Oral ENS Ulm, 36 planches - Page 4 — Les-mathematiques.net The most powerful custom community solution in the world

Oral ENS Ulm, 36 planches

124

Réponses

  • @side:
    Soit $\left(\Omega,\mathcal{T},\mathbb{P}\right)$ un espace probabilisé.
    Soit $X$ une variable aléatoire définie sur $\left(\Omega,\mathcal{T},\mathbb{P}\right)$.
    On note $F_X$ la fonction de répartition de $X$.
    Alors:
    1) Pour tout $x\in\mathbb{R}$, $F_X$ est continue à droite en $x$.
    2) Pour tout $x\in\mathbb{R}$, $\mathbb{P}(X=x) = F_X(x) -\lim\limits_{t\to x^-}F_X(t)$.

    1) 2) se démontrent en utilisant notamment les théorèmes de limite monotone dans le contexte des probabilités ("continuité croissante/décroissante") et à partir de ces deux points, tu montres facilement que:

    3) Pour tout $x\in\mathbb{R}$, $F_X$ est continue en $x$ si et seulement si $\mathbb{P}(X=x)=0$.
  • Un (petit) début pour l'exercice 28.1.

    Traitons le cas $\lambda = 0$, alors le nombre d'entiers compris entre $10^n$ et $10^{n+1}-1$ est $9^n$ et chacun des termes de la série est majoré par $1/10^n$, et comme $\sum (9/10)^n$ converge on a le résultat.

    Pour la suite je ne parviens pas à conclure. Je pense qu'on peut essayer en écrivant $S_{\lambda}=\sum_{n\ge 0} \sum_{k=0}^{\lambda \log _{10} (n)}S(n,k)$ où $S(n,k)$ est la somme des $1/a$ où $a$ a $n$ chiffres et s'écrit avec exactement $k$ fois le chiffre $9$. Mais je ne sais pas évaluer $S(n,k)$ ...
  • @Side (mais un peu trop tard...)

    -La série converge à réalisation fixée (il y a convergence simple) et donc la série définit bien une VA (convergence simple d'une suite de fonctions mesurables).

    -La loi de $X$ est sans atomes en $x$ équivaut à $F_{X}$ est continue en $x$ est un résultat que l'on peut supposer connu (mais suppose que l'on sait qu'une fonction croissante est continue en $x$ si et seulement si $f(x^{+})=f(x^{-}).$

    -Le point $2$ est faux... Justement $E$ n'est jamais dénombrable (il s'agit d'un ensemble de Cantor pour $\alpha\neq \frac{1}{2}$)!
    Avec les notations de l'énoncé : si par exemple $\alpha=\frac{1}{2},$ le support de la loi de la série est $[-2,2]$ et cette loi est la mesure de Lebesgue (c'est un exercice "amusant" de le montrer mais ce n'est pas nécessaire pour répondre à l'exercice)

    -Enfin, on peut optimiser la preuve que j'ai donnée pour la faire fonctionner pour tout $\alpha<\frac{1}{2}$ (et pas seulement dans un voisinage de $0$).
    Avec mes notations, il suffit de prouver l'inclusion $\displaystyle A_{l}\subset \bigcap_{k=0}^{l+1-B}\{X_{k}=\varepsilon_{k}\}$ où $B$ est un entier suffisamment grand de manière à avoir $\displaystyle\alpha^{B}\leq \varepsilon$ alors que $\displaystyle \alpha\leq \frac{1-\varepsilon}{2}.$
  • Planche 25 (premier exercice)

    *On montre facilement par récurrence sur $k\in\mathbb{N}$ que pour tout $x\in[0,1],$ $P_{k}(x),Q_{k}(x)\in[0,1]$
    (et on a également pour tout $k\geq 1,$ $P_{k}(0)=1;Q_{k}(0)=0$).

    **Soit $k\geq 0.$ Considérons $x\in[0,1].$ On a alors en utilisant les relations de récurrence
    $$P_{k+1}(x)=\left(1-x+\int_{0}^{x}Q_{k}(t)dt\right)=\left(1-\int_{0}^{x}\left(1-\int_{0}^{t}P_{k}(u)du\right)^{2}dt\right)^{2}.$$

    -On a ainsi en utilisant une identité remarquable (et comme les fonctions $P_{k}$ sont à valeurs dans $[0,1]$) :
    \begin{align*}
    \vert P_{k+2}(x)-P_{k+1}(x)\vert & = \left( 2-\int_{0}^{x}\left[ \left( 1-\int_{0}^{t}P_{k+1}(u)du \right)^{2}+\left( 1-\int_{0}^{t}P_{k+1}(u)du \right)^{2}\right]dt\right)\\
    & \mbox{ }\mbox{ }\mbox{ }\mbox{ }\mbox{ }\times \left\vert\int_{0}^{x}\left[ \left( 1-\int_{0}^{t}P_{k+1}(u)du\right)^{2}-\left(1-\int_{0}^{t}P_{k+1}(u)du \right)^{2}\right]dt\right\vert\\
    & \leq 2\int_{0}^{x}\left( 2-\int_{0}^{t}P_{k+1}(u)du-\int_{0}^{t}P_{k+1}(u)du\right)dt\times \int_{0}^{x}\left(\int_{0}^{t}\big\vert P_{k+1}(u)-P_{k}(u)\big\vert du\right)dt \\
    & \mbox{ en appliquant l'inégalité triangulaire intégrale}\\
    & \leq 4\int_{0}^{x}\left(\int_{0}^{t}\big\vert P_{k+1}(u)-P_{k}(u)\big\vert du\right)dt \mbox{ (*) }.
    \end{align*}

    -De même, on a également en écrivant $\displaystyle x=\int_{0}^{x}1dt,$
    \begin{align*}
    \vert P_{k+1}(x)+Q_{k+1}(x)-1\vert & = \big\vert \int_{0}^{x}\left(P_{k}(t)+Q_{k}(t)-1\right)dt \big\vert \times \left(2-x+\int_{0}^{x}(Q_{k}(t)-P_{k}(t))dt\right)\\
    & \leq \left(2-x+\int_{0}^{x}\big\vert Q_{k}(t)-P_{k}(t)\big\vert dt\right)\times \int_{0}^{x}\big\vert P_{k}(t)+Q_{k}(t)-1\big\vert dt\\
    & \mbox{ en appliquant l'inégalité triangulaire intégrale}\\
    & \leq 2\int_{0}^{x}\big\vert P_{k}(t)+Q_{k}(t)-1\big\vert dt \mbox{ (**)}.
    \end{align*}

    -On montre alors (grâce aux formules $(*)$ et $(**)$) par récurrence sur $k\geq 0,$ $$\forall t\in[0,1] :\mbox{ } \vert P_{k+1}(t)-P_{k}(t)\vert\leq \frac{(2t)^{2k}}{(2k)!} \mbox{ et } \vert P_{k}(t)+Q_{k}(t)-1\vert \leq \frac{(2t)^{k}}{k!}.$$

    Les initialisations sont claires car $\displaystyle P_{0}=0=Q_{0} \mbox{ et } P_{1}(t)=(1-t)^{2}\leq 1.$

    Enfin par $(*),$ on a en intégrant en cascade :
    \begin{align*}
    \vert P_{k+2}(x)-P_{k+1}(x)\vert & \leq 4\int_{0}^{x}\left(\int_{0}^{t}\frac{(2u)^{2k}}{(2k)!}du\right)dt\\
    & \leq \frac{4\times 2^{2k}}{(2k+1)!}\int_{0}^{x}t^{2k+1}dt\\
    & \leq \frac{(2t)^{2k+2}}{(2k+2)!}.
    \end{align*}

    Et par $(**),$ on a en intégrant :
    \begin{align*}
    \vert P_{k+1}(x)+Q_{k+1}(x)-1\vert & \leq 2\int_{0}^{x}\frac{(2t)^{k}}{k!}dt\\
    & \leq \frac{(2x)^{k+1}}{(k+1)!}.
    \end{align*}

    Ainsi, les séries de terme général respectifs $\|P_{k}-P_{k+1}\|_{\infty,[0,1]}$ et $\|P_{k+1}+Q_{k+1}-1\|_{\infty,[0,1]}$ sont convergentes et donc la suite $(P_{k})$ converge uniformément vers $P$ sur $[0,1]$ et la suite $(Q_{k})$ converge uniformément vers $Q$ sur $[0,1]$ (car $\displaystyle \left(\mathcal{C}^{0}([0,1],\mathbb{R}),\|.\|_{\infty,[0,1]}\right)$ est un espace de Banach).

    De plus, on a $\displaystyle P+Q=1$ sur $[0,1].$

    En passant à la limite, il vient également pour tout $x\in[0,1]$ : $\displaystyle P(x)=\left(1-x+\int_{0}^{x}Q(t)dt\right)^{2} \mbox{ et } Q(x)=1-\left(1-\int_{0}^{x}P(t)dt\right)^{2}.$

    Remarque : Plutôt que d'utiliser un argument de complétude, on peut dans le programme de prépa remarquer la convergence absolue des séries en jeu à $x$ fixé. Ainsi, il y a convergence simple des suites $(P_{k})$ et $(Q_{k})$ en tout point de $[0,1].$ Les deux dernières relations fonctionnelles s'obtiennent par convergence dominée car les suites de fonctions en jeu sont bornées uniformément, ce qui est une domination valable sur un segment.

    ***La limite cherchée est alors (en utilisant $P+Q=1$ et en passant à la limite) $\displaystyle I=\int_{0}^{1}\left(1-x+\int_{0}^{x}Q(t)dt\right)^{3}=\int_{0}^{1}\left(1-\int_{0}^{x}P(t)dt\right)^{3}.$

    Il ne reste plus qu'à déterminer $\displaystyle \phi(x)=\int_{0}^{x}P(t)dt.$
    Or, on sait que $\displaystyle P(x)=\left(1-\int_{0}^{x}P(t)dt\right)^{2} \mbox{ i.e. } \phi'(x)=\left(1-\phi(x)\right)^{2}.$
    En séparant les variables et en utilisant $\phi(0)=0,$ il vient $\displaystyle \frac{1}{1-\phi(x)}-1=x \mbox{ i.e. } \phi(x)=1-\frac{1}{1+x}=\frac{x}{1+x}$ (d'où $\displaystyle P(x)=\frac{1}{(1+x)^{2}}$).

    Ainsi, on obtient finalement $\displaystyle I=\int_{0}^{1}\frac{dx}{(1+x)^{3}}=\left[\frac{-1}{2(1+x)^{2}}\right]_{0}^{1}=\frac{3}{8}.$
  • Merci à BobbyJoe pour toutes ces corrections !
  • 8.1 n'est pas facile... je suppose que $b_n = \mathbb{E}(M_n)$ où $M_n = \max_{1 \leq i \leq n} X_i$ et que donc la condition revient à montrer que $\mathrm{Var}(M_n) / (\mathbb{E}(M_n))^2)$ tend vers $0$ sous la condition $1$.

    8.2 a l'air nettement moins redoutable : $X^2 - X + 1$ est alors le polynôme minimal de $A$ donc $\mathrm{tr}(A) = \det(A) = 1$, et finalement, $r$ étant fixé au départ, il faut montrer l'existence de deux rationnels $a$ et $b$ tels que $a(1-a) - b(r-1-b) = 1$.
  • @ShumiSutil effectivement le 8.2 à été fait par la même méthode et plutôt facile pour un exo de ce niveau. Pas comme le 8.1.
  • Toujours en conjecturant que $b_n = \mathbb(E_n)$ (qui tend bien vers l'infini) et en notant $p(x) = \mathbb{P}(X_1 \geqslant x)$, il faut alors montrer que $$

    \forall a > 1, \ \frac{p(ax)}{p(x)} \to 0,

    $$ si, et seulement si, $$

    \mathbb{E}(M_n^2) \sim \mathbb{E}(M_n) ^2,

    $$ soit en utilisant $E(Y^p) = p \int_0^{+\infty} t^{p-1} \mathbb{P} (Y \geqslant t) dt = p \int_0^{+\infty} t^{p-1} (1 - \mathbb{P} (Y < t)) dt$
    \begin{align*}
    \mathbb{E}(M_n^2) &= 2\int_0^{\infty} x( 1 - \mathbb{P} (M_n < x)) dx \\
    &= \int_0^{\infty} x ( 1 - \mathbb{P}^n (X_1< x)) dx \\
    &= 2 \int_0^{\infty} x ( 1 - (1- p(x))^n) dx \\
    & \sim \bigg( \int_0^{\infty} ( 1 - (1- p(x))^n) dx \bigg)^2
    \end{align*}
  • Bonjour,
    @BobbyJoe, il y a quelque chose que je n'ai pas compris dans votre preuve de la planche 23. Peut-on remplacer le deuxième point de votre démonstration par l'argument suivant, ou ai-je fait une erreur ? (Je reprend vos notations)
    Soit $0 < \alpha < 1/2$. Soit $t = \sum_{k\geq 0}\alpha^{k}\varepsilon_{k} \in K_\alpha$, cette écriture étant unique. $\mathbb{P}(Z_\alpha = t) = \mathbb{P}(\forall k, X_k = \varepsilon_{k}) = 0$. Donc $\displaystyle\lim_{x \rightarrow t^+} F_\alpha(x) - \lim_{x \rightarrow t^-} F_\alpha(x) = \mathbb{P}(Z_\alpha = t) = 0$ pour tout $t \in \mathbb{R}$ et $F_\alpha$ est continue.
    Merci d'avance
  • @Mickaël et SchumiSutil : une idée pour le 8.1 est de remarquer que $
    P\left(\max_{1\leqslant i\leqslant n} X_i < t\right) = P(X_1 < t)^n$. Ceci impose de choisir $b_n$ de sorte que $P(X_1 \geqslant b_n)$ soit de l'ordre de $\frac1n$. Reste à vérifier que ceci fonctionne bien si et seulement si la condition (1) est satisfaite.
  • Mickaël : Planche 28 Exercice 1.

    La méthode la plus directe est probablement la même que celle que tu as utilisé pour $\lambda=0$. On compte les entiers de $k$ chiffre s'écrivant avec moins de $k\lambda$ fois le chiffre $9$, si leur proportion tend vers une quantité strictement positive la série diverge, si leur proportion tend vers $0$ assez vite alors la série converge. Je n'ai pas fais les calculs parce que ça me semble pénible mais je suppose qu'avec des estimations un peu astucieuses ça se fait.

    Autre approche :

    Un résultat bien connu nous dit que pour presque tout $x\in[0;1]$ la proportion de $9$ dans les décimales de $x$ est exactement 1/10. Ceci permet d'en déduire que pour tout $\lambda >1/10$ la proportion d'entiers de $k$ chiffres s'écrivant avec moins de $\lambda k $ chiffre $9$ tend vers $1$. Une minoration du même type que ce que tu as fais pour le cas $\lambda=0$ montre alors que la série diverge.

    Pour $\lambda \in ]0;1/10]$ c'est plus compliqué, on sait grâce au résultat bien connu précédent que la proportion va tendre vers $0$ mais on ne sait rien de la vitesse de convergence, impossible de conclure. Heureusement pour nous, il existe d'autres théorèmes bien connus en mathématiques. Oublions un instant qu'un nombre ne devrait pas commencer par le chiffre $0$ (sauf s'il ce nombre est $0$), tirer un nombre à $k$ chiffre au hasard revient alors à tirer au hasard de façon indépendantes $k$ chiffres entre $0$ et $9$. Le nombre de $9$ dans un entier à $k$ chiffres tiré au hasard est alors une variable aléatoire qui s'exprime comme une somme de $k$ variables aléatoires indépendantes de loi de Bernoulli de paramètre $1/10$. Notons $B_1,\ldots , B_k$ ces variables aléatoires, le théorème central limite nous dit que
    \[
    \mathbb P\left(\sum_{i=1}^k B_i \leq t \frac{9}{100} \sqrt k +\frac{k}{10} \right) \underset{k\to \infty}{\to} \Phi(t)
    \]
    où $\Phi$ est la fonction de répartition de la loi normale centrée réduite. Pour $t=0$ on retrouve le cas critique $\lambda=1/10$, on voit que la proportion limite est $1/2$, la série est donc divergente. Il faut travailler un poil plus si on veut une preuve complète qui prenne en compte le comportement particulier du premier chiffre mais le résultat est le même et la méthode marche aussi pour $\lambda>1/10$.

    Pour le cas $\lambda<1/10 $ le TCL ne permet a priori pas de conclure, même en utilisant l'Inégalité de Berry-Esseen... en tout cas je n'ai pas trouvé comment faire. Je n'ai pas trouvé d'idée astucieuse qui épargnerai un calcul à la main mais ça doit exister. En tout cas je pense que la série converge pour tout $\lambda<1/10$. Si on note $p_k$ la proportion d'entiers à $k$ chiffres ayant moins de $k\lambda$ dans leur écriture on peut majorer la série par quelque chose d'équivalent à
    \[
    \sum_{k=1}^\infty \log((1+9p_k)10^k )-\log(10^k) = \sum_{k=1}^\infty \log(1+9p_k)
    \]
    qui converge ssi $\sum pk$ converge. J'aurais tendance à dire que pour $\lambda<1/10$ la suite $(p_k)$ doit décroître à une vitesse comprise entre la décroissance géométrique et la décroissance polynomiale.
  • @Calli
    Oui, tu as raison et c'est plus simple si on a prouvé l'unicité d'écriture...

    En revanche, on peut sauter cette étape et faire comme j'ai fait (sans ce soucier de l'unicité d'écriture).
    J'avoue ne pas avoir optimisé la preuve et j'ai consigné ce que me semblait intéressant.

    Ceci a le mérite de montrer en plus que $F_{\alpha}$ est höldérienne d'exposant $\displaystyle \frac{\ln(2)}{\ln(\frac{1}{\alpha})}$ si $\alpha\in\,]0,\frac{1}{2}[$ (en fait, ceci est valable pour $\alpha=\frac{1}{2}$ mais par une autre méthode).
  • Planche 8 (Exercice 1)

    *$\boxed{i)\Longrightarrow ii)}$

    -On construit une suite strictement croissante $(b_{n})_{n\in\mathbb{N}^{*}}$ tendant vers l'infini et qui vérifie pour tout $n\geq 1$ : $\displaystyle \mathbb{P}(X\geq b_{n+1})\leq \frac{1}{n} \mbox{ et }\mathbb{P}(X\geq b_{n})>\frac{1}{n}.$

    Soit alors $a>1.$ On choisit $a$ proche de $1$ de manière à pouvoir écrire $a\sim1+\varepsilon$ et $\frac{1}{a}\sim 1-\varepsilon$ où $\varepsilon>0$ est assez petit.

    Par $i),$ on a alors $\displaystyle \frac{\mathbb{P}(X\geq ab_{n+1})}{\mathbb{P}(X\geq b_{n+1})}\longrightarrow 0 \mbox{ et } \frac{\mathbb{P}(X\geq b_{n})}{\mathbb{P}(X\geq \frac{b_{n}}{a})}\longrightarrow 0.$

    On en déduit alors :
    \begin{align*}
    n\mathbb{P}(X\geq ab_{n+1}) & = \frac{\mathbb{P}(X\geq ab_{n+1})}{\mathbb{P}(X\geq b_{n+1})}\times n\mathbb{P}(X\geq b_{n+1})\\
    & \leq \frac{\mathbb{P}(X\geq ab_{n+1})}{\mathbb{P}(X\geq b_{n+1})} \longrightarrow 0.\\
    \mbox{ et, } \frac{1}{n\mathbb{P}(X\geq \frac{b_{n}}{a})} & = \frac{\mathbb{P}(X\geq b_{n})}{\mathbb{P}(X\geq \frac{b_{n}}{a})}\times \frac{1}{n\mathbb{P}(X\geq b_{n})}\\
    & \leq \frac{\mathbb{P}(X\geq b_{n})}{\mathbb{P}(X\geq \frac{b_{n}}{a})} \longrightarrow 0\\
    & \mbox{ i.e. } n\mathbb{P}(X\geq \frac{b_{n}}{a}) \longrightarrow +\infty.
    \end{align*}

    -On a alors en notons pour $n\geq 1,$ $\displaystyle M_{n}=\max_{k=1,\ldots,n}X_{k}$ :
    \begin{align*}
    \mathbb{P}(\frac{b_{n}}{a}< M_{n} \leq ab_{n}) & = \mathbb{P}(M_{n}\leq ab_{n})-\mathbb{P}(M_{n}\leq \frac{b_{n}}{a})\\
    & =\exp\left[ n\ln \big(1-\mathbb{P}(X\geq ab_{n})\big) \right]-\exp\left[ n\ln \left(1-\mathbb{P}(X\geq \frac{b_{n}}{a})\right) \right] \mbox{ par indépendance des va en jeu}\\
    & \longrightarrow 1-0=1\\
    & \mbox{ car } n\ln\left( 1-\mathbb{P}(X\geq ab_{n}\right)\sim -n\mathbb{P}(X\geq ab_{n})\longrightarrow 0 \mbox{ et } n\ln \left (1-\mathbb{P}(X\geq \frac{b_{n}}{a})\right)\sim -n\mathbb{P}(X\geq \frac{b_{n}}{a})\longrightarrow -\infty.
    \end{align*}

    Ainsi, l'implication désirée est démontrée.

    **$\boxed{ii)\Longrightarrow i)}$

    -Il suffit de démontrer $i)$ pour tout $a>1$ proche de $1$ car alors si $b>a,$ $i)$ est clairement satisfaite pour $b$ également.

    -Soit $a>1$ proche de $1.$

    En supposant $ii),$ on a d'une part $\displaystyle 1-\left(1-\mathbb{P}(X\geq ab_{n})\right)^{n}=\mathbb{P}(M_{n}\geq ab_{n})\longrightarrow 0.$
    Mais alors, $\displaystyle -n\mathbb{P}(X\geq ab_{n})\longrightarrow 0.$
    Au pire, si vraiment on veut le prouver : cette suite est minorée sinon on a une contradiction. On prend alors une valeur d'adhérence. Cette suite bornée n'en a qu'une seule, à savoir $0.$

    D'autre part, toujours en utilisant $ii),$ on a $\displaystyle \left(1-\mathbb{P}(X\geq \frac{b_{n}}{a})\right)^{n}=\mathbb{P}(M_{n}\leq \frac{b_{n}}{a})\longrightarrow 0.$
    On a ainsi $\displaystyle -n\mathbb{P}(X\geq \frac{b_{n}}{a})\longrightarrow -\infty.$

    -On procède finalement par encadrement en utilisant la décroissance de la fonction de survie.
    Soit $\varepsilon>0$ tel que $\varepsilon\ll 1.$ On a alors pour tout $n\gg 1$ : $\displaystyle \mathbb{P}(X\geq ab_{n})\leq \frac{\varepsilon}{n} \mbox{ et } \mathbb{P}(X\geq \frac{b_{n+1}}{a})\geq \frac{1}{n+1}\geq \frac{1-\varepsilon}{n}.$

    Soit alors $x\in\mathbb{R}^{+}$ tel que $\displaystyle \frac{b_{n}}{a}\leq x\leq \frac{b_{n+1}}{a}.$

    On a alors par décroissance de la fonction de survie :
    \begin{align*}
    \mathbb{P}(X\geq a^{2}x) & \leq \mathbb{P}(X\geq ab_{n})\\
    & \leq \frac{\varepsilon}{n}\\
    & \leq \frac{\varepsilon}{1-\varepsilon}\mathbb{P}(X\geq \frac{b_{n+1}}{a})\\
    & \leq \frac{\varepsilon}{1-\varepsilon}\mathbb{P}(X\geq x).
    \end{align*}

    L'implication désirée est démontrée (quitte à redéfinir certains paramètres...).
  • Voici une correction plus "self-contained" (j'ai sauté certains calculs...) de l'exercice $30.$

    Remarque : je n'ai pas trouvé de démonstration astucieuse! Le résultat est assez fin (car optimal)... La preuve proposée ici bien que technique est naturelle (on procède par réduction et on pousse le crayon jusqu'à temps de résoudre un problème purement technique à la fin) et quantitative (si on fait bien attention, on peut d'estimer le $n$ nécessaire pour réaliser la conclusion désirée).

    $i)$ *Un polynôme qui satisfait les hypothèses de l'énoncé est réel.
    En effet, soit on utilise des polynômes d"interpolation en des points de $\mathbb{R}^{+}$ soit, comme une limite de fonctions réelles est réelle, les dérivées du polynôme sont réelles sur $\mathbb{R}^{+}$ et une utilisation des formules de Taylor permet de conclure.

    **On procède à la réduction suivante :

    Un polynôme $P$ qui satisfait les hypothèses de l'énoncé se factorise de la manière suivante :
    $$P(X)=\lambda\prod_{i\in I}(X+a_{i})^{\alpha_{i}}\times \prod_{j\in J}(X^{2}+b_{j}+c_{j})^{\beta_{j}}$$
    où $\lambda>0$ (prendre un équivalent de $P$ en $+\infty$) et pour tout $i\in I,$ $a_{i}>0$ (ce sont les racines réelles de $P$ qui vivent dans $\mathbb{R}^{-*})$ et pour tout $j\in J,$ $b_{j}^{2}-4c_{j}<0$ avec $c_{j}>0.$
    Comme un produit de polynômes à coefficients strictement positifs est encore à coefficients strictement positifs, il suffit de traiter le cas d'un polynôme $P(X)=X^{2}+aX+b$ vérifiant $a^{2}-4b<0$ et $b>0.$

    On suppose $P$ de cette forme jusqu'à la fin de cet exercice.

    ***-Le cas $a\geq 0$ est direct car alors $(1+X)P(X)$ est à coefficients strictement positifs.

    -On traite alors le cas $a<0.$

    Il existe $\varepsilon>0$ (avec $\varepsilon\ll 1$) tel que $-a<2\sqrt{b}(1-\varepsilon).$

    But : On veut trouver $n\gg 1$ tel que $\displaystyle (1+X)^{n}P(X)=\sum_{k=0}^{n+2}b_{k}X^{k}$ où pour tout $k\in\{0,\ldots,n+2\},$ $b_{k}>0.$

    Cette dernière condition s'écrit également $\displaystyle \forall k\in \{0,\ldots,n+2\},\mbox{ } -a\binom{n}{k-1}<\binom{n}{k-2}+b\binom{n}{k}.$

    Pour $k\in\{0,1,n+1,n+2\},$ on doit vérifier $b>0;\mbox{ }nb>-a;\mbox{ }n>-a \mbox{ et } 1>0$ (ce qui est manifestement vrai si $n\gg 1$ vu que $b>0$).

    Les conditions restantes s'écrivent (après avoir simplifié les coefficients binomiaux et arrangé un peu l'expression) pour $k\in\{2,\ldots,n\},$ $\displaystyle -a<(n+1)\left( \frac{b}{k}+\frac{1}{n+2-k}\right)-(b+1):=\phi(k).$

    -Montrons donc qu'il existe $n\gg1$ tel que pour tout $x\in[2,n],$ $\displaystyle \phi(x)\geq 2\sqrt{b}(1-\varepsilon).$

    On a tout d'abord $\displaystyle \forall x\in[2,n],\mbox{ } \phi'(x)=(n+1)\frac{\left( x-\sqrt{b}(n+2-x)\right)\times \left( x+\sqrt{b}(n+2-x)\right) } {x^{2}(n+2-x)^{2}}.$

    Notons $\displaystyle x_{1}=\frac{(n+2)\sqrt{b}}{1+\sqrt{b}}\in[2,n]$ (si $n\gg1,$ plus précisément si $n\geq 2\sqrt{b}$) et $\displaystyle x_{2}=\frac{(n+2)\sqrt{b}}{\sqrt{b}-1}$ si $b\neq 1.$

    Avec ces notations réduites, il vient pour tout $x\in[2,n]$ : $$\phi'(x)=\frac{(1-b^{2})(n+1)(x-x_{1})(x-x_{2})}{x^{2}(n+2-x)^{2}} \mbox{ si } b\neq 1 \mbox{ et } \phi'(x)=\frac{2(n+1)(n+2)(x-x_{1})}{x^{2}(n+2-x)^{2}} \mbox{ si } b=1.$$

    En traitant les trois cas séparément : $\boxed{0<b<1}$ (i.e. $x_{2}<0$ et $1-b^{2}>0$), $\boxed{ b=1} $ et $\boxed{b> 1}$ (i.e. $x_{2}>n$ et $1-b^{2}<0$), on montre que $\phi$ atteint son minimum en le point $x_{1}.$ Ce minimum est : $\displaystyle \phi(x_{1})=\frac{(n+1)(1+\sqrt{b})^{2}}{n+2}-(1+b)\longrightarrow 2\sqrt{b}.$

    Ainsi, pour $n\gg 1,$ on a bien pour tout $k\in\{2,\ldots,n\},$ $\displaystyle \phi(k)\geq \phi(x_{1}) \geq 2\sqrt{b}(1-\varepsilon)>-a.$

    Toutes les conditions recherchées sont alors satisfaites dès que $n$ est choisi assez grand et la preuve du premier point est ainsi achevée.

    ****Remarque : Le résultat suivant découle directement de ce que l'on vient de prouver. Si $P$ est strictement positif sur $\mathbb{R}^{+*}$ alors il existe $n\in\mathbb{N}$ tel que $(1+X)^{n}P(X)$ est à coefficients positifs.
    On pourrait s'en servir pour traiter le deuxième point de l'exercice...

    $ii)$ Le deuxième point découle de la première partie par un changement de variables (homographie).

    *Soit $P$ un polynôme strictement positif sur $[-1,1].$ Notons $n\geq 1$ le degré de $P$ (si $P$ est constant, le résultat est clair!).
    On considère alors le polynôme $\displaystyle Q(X)=(1+X)^{n}P\left( \frac{1-X}{1+X} \right)$ est de degré $n$ et est strictement positif sur $\mathbb{R}^{+}$ (car l'image de $\mathbb{R}^{+}$ par l'homographie est $]-1,1],$ ensemble sur lequel $P$ est strictement positif).
    Par le point $i),$ il existe alors $N\in\mathbb{N}$ tel que $\displaystyle (1+X)^{N}Q(X)=(1+X)^{n+N}P\left( \frac{1-X}{1+X} \right)=\sum_{k=0}^{n+N}a_{k}X^{k}$ où pour tout $k\in\{0,\ldots,n+N\},$ $a_{k}>0.$
    Ainsi, pour tout $x\in ]-1,1],$ on a $\displaystyle \left(\frac{2}{1+x}\right)^{n+N}P(x)=\sum_{k=0}^{n+N}a_{k}(1-x)^{k}(1+x)^{-k}.$
    On a alors pour tout $x\in]-1,1],$ $\displaystyle P(x)=\sum_{k=0}^{n+N}\frac{a_{k}}{2^{n+N}}(1-x)^{k}(1+x)^{n+N-k}.$
    Ces deux polynômes coïncidant sur une partie infinie, ils sont égaux et on a obtenu la décomposition voulue dans ce cas.

    **Soit $P$ un polynôme strictement positif sur $]-1,1[.$
    Par continuité, on a $\displaystyle P(-1)\geq 0 \mbox{ et } P(1)\geq 0.$ En factorisant $P$ par les racines éventuelles $-1$ et $1,$ on peut écrire : $\displaystyle P(X)=(1-X)^{\alpha}(1+X)^{\beta}Q(X)$ où $Q$ est strictement positif sur $[-1,1].$
    Ainsi, en appliquant la première étape, $P$ se décompose bien comme demandé mais ici, les coefficients de la décomposition sont seulement positifs (éventuellement nuls).
  • Une solution astucieuse de l'exercice 3089124
    Le 😄 Farceur


  • Exercice 9.1. [Communication personnelle] Déjà pour les bornes on se convainc que le maximum atteignable est satisfait lorsque $n_1$ est maximal, puis $n_2$ minimal, etc, c'est à dire en $b:=|20,5,20,5, \dots]$ et le minimum en $a:=[5,20,5,20,\dots]$. Déjà on peut calculer, au moins formellement les valeurs de $a$ et $b$, celle-ci satisfont par définition $b=2+\frac{20}{2+\frac{5}{b}}$ et $a=2+\frac{5}{2+\frac{20}{a}}$. On trouve donc $a=5/2$ et $b=10$. Affirmation : $A=[5/2,10]$. Pour le voir il suffit de :

    (1) Montrer effectivement que $a$ et $b$ convergent, c'est à dire, par exemple, étudier la suite $a_{n+1}=2+\frac{20}{2+\frac{5}{a_n}}$.

    (2) Remarquer que les fonctions $x \mapsto 2+\frac{20}{x}$ et $x \mapsto 2+\frac{5}{x}$ envoient le segment $[5/2,10]$ sur les segments $[5/2,4]$ et $[4,10]$ de sorte que pour $x\in [5/2,10]$ fixé, on puisse construire une suite $(x_n)$ par dichotomie telle que $x=[x_1, x_2, \dots]$.
  • $@ Poirot $

    Jolie livre
  • $@Poirot$

    Bon livre comme d'habitude (tu)
  • side écrivait : http://www.les-mathematiques.net/phorum/read.php?4,1841908,1842144#msg-1842144
    [Inutile de recopier un message présent sur le forum. Un lien suffit. AD]
    @side acceptes-tu de poster la démonstration qui donne cet équivalent ?
  • J'ai reçu plusieurs messages me demandant où se trouve la solution de l'exercice 7. Il se trouve que side l'a traité dans le fil suivant : http://www.les-mathematiques.net/phorum/read.php?4,1839904,1840016#msg-1840016 il n'y a pas longtemps.

    Concernant l'exercice 28.1, ce que dit Corto fonctionne bien pour $\lambda \geqslant 1/10$. Ensuite pour ce qui est du cas $\lambda <1/10$ on peut effectivement compter comme dans le cas $\lambda=0$. Disons que on accepte qu'un nombre commence par un zéro histoire d'avoir des calculs simples. Notons $u_n$ la somme des $1/j$ où $j$ a $10^n$ chiffres dont moins de $\lambda n$ chiffres $9$. Il s'agit de montrer la convergence de $\sum u_n$. Pour cela on minore tous les $j$ par $10^n$ et on a donc $$u_n\leq 10^{-n} \sum_{k=0}^{\lfloor \lambda n\rfloor } \binom{n}{k}9^{n-k}\,.$$ Ensuite on observe que pour $n$ suffisamment grand et parce que $\lambda < 1/10$, la suite $\left(\binom{n}{k}9^{n-k}\right)_{0\leq k \leq \lfloor \lambda n \rfloor}$ est croissante et donc $|u_n| \lesssim n \binom{n}{\lfloor \lambda n \rfloor}10^{-n}9^{(1-\lambda)n}$, ce qui s'écrit avec Stirling, $|u_n| \lesssim n^{1/2}\exp (cn)$ où $c=-\log 10 +(1-\lambda) \log 9 -\lambda \log \lambda -(1-\lambda)\log (1-\lambda)<0$ pour $\lambda \in ]0,1/10[$ et on a d'ailleurs $c=0$ pour $\lambda=1/10$.

    Pour avoir une vraie preuve il faudrait tenir compte du fait que le premier chiffre d'un nombre à $n$ chiffres ne peut être un $0$ et donc on a plutôt $$u_n=\sum_{k\leq n\lambda}\binom{n-1}{k}8\times 9^{n-1-k}+\sum_{k\leq n\lambda}\binom{n-1}{k-1}9^{n-k}\,.$$
  • Pour la planche 28 : Exercice 1.
    Tout d'abord remercions @Corto pour son excellente idée (ce qui lui manquait pour conclure était l'inégalité de concentration appelée inégalité d'Hoeffding, cf th.4 du lien suivant : http://cs229.stanford.edu/extra-notes/hoeffding.pdf) et je tiens également à remercier @Mickäel pour les discussions que j'ai eues avec lui hors forum.

    -Tout d'abord introduisons quelques notations.

    Notons pour $k\in\{0,\ldots,9\},$ et $n\geq 1$ un entier, $\displaystyle a_{k}(n)$ l'occurence du chiffre $k$ dans l'écriture en base $10$ de l'entier $n$ et $b_{n}$ le nombre de chiffres constituant l'écriture en base $10$ de l'entier $n.$
    On note alors pour $\lambda\in]0,1[,$ $\displaystyle A_{\lambda}=\{k\geq 1\mbox{ }|\mbox{ }a_{9}(k)\leq \lambda b(k)\}.$
    On introduit alors pour $n\in \mathbb{N}$ : $\displaystyle S_{n,\lambda}=\left\{ k\in\{10^{n},\ldots,10^{n+1}-1\}\mbox{ }|\mbox{ } k\in A_{\lambda} \right\}=\left\{ k\in\{10^{n},\ldots,10^{n+1}-1\}\mbox{ }|\mbox{ } a_{9}(k)\leq \lambda n\right\}.$
    Ainsi, on a $\displaystyle \sum_{n\geq 0}\frac{\# S_{n,\lambda}}{10^{n+1}}\leq \sum_{k\in A_{\lambda}}\frac{1}{k}\leq \sum_{n\geq 0}\frac{\# S_{n,\lambda}}{10^{n}}.$

    -On dénombre alors (en discutant suivant que $9$ soit placé en tête du nombre ou non) et on obtient pour $n\geq 1$ : $\displaystyle \# S_{n,\lambda}=\sum_{k\leq \lambda n}\binom{n-1}{k}8\times 9^{n-1-k} + \sum_{k\leq \lambda n}\binom{n-1}{k-1}9^{n-k}.$

    En utilisant la formule du triangle de Pascal (et à défaut d'une suite géométrique convergente près), on a ainsi pour $n\geq 1$ (pour des constantes absolues) : $\displaystyle \frac{\# S_{n,\lambda}}{10^{n}}\approx \sum_{k\leq \lambda n}\binom{n}{k}(\frac{1}{10})^{k}(\frac{9}{10})^{n-k}:=B_{n,\lambda}.$

    -Considérons alors $(X_{i})_{i\geq 1}$ une famille de Bernoulli i.i.d, de paramètre $\frac{1}{10}.$
    On a ainsi pour $n\geq 1,$ $\displaystyle B_{n,\lambda}=\mathbb{P}(\sum_{i=1}^{n}X_{i} \leq n\lambda).$

    On a deux cas :
    i) ou $\displaystyle \lambda\geq \frac{1}{10}.$

    On a $\displaystyle B_{n,\lambda}=\mathbb{P}\left( \frac{1}{\sqrt{n}}\sum_{i=1}^{n}\left( X_{i}-\mathbb{E}[X_{i}]\right)\leq (\lambda-\frac{1}{10})\sqrt{n}\right)$ et une application du TCL donne alors que $\displaystyle \lim_{n\rightarrow +\infty} B_{n,\lambda}\geq \frac{1}{2}.$
    Ainsi, la série considérée diverge (cependant, une simple estimée via l'inégalité de Tchebytchev permet également de conclure à la divergence).

    ii) ou $\displaystyle 0<\lambda<\frac{1}{10}.$

    On a $\displaystyle B_{n,\lambda}=\mathbb{P}\left( \frac{1}{n}\sum_{i=1}^{n}\left( X_{i}-\mathbb{E}[X_{i}]\right)\leq -(\frac{1}{10}-\lambda)\right)$ et une application de l'inégalité d'Hoeffding (unilatérale) donne $B_{n,\lambda}\leq e^{-2\varepsilon^{2}n}$ où $\displaystyle \varepsilon:=\frac{1}{10}-\lambda>0.$
    Ainsi, la série considérée converge (cependant, une simple estimée via Stirling permet également de conclure comme Mickaël l'a indiqué, au prix de quelques calculs supplémentaires et d'une perte polynomiale...).
  • Je reviens sur la planche 34 comme promis.
    Mickaël a écrit:
    Sauf que n=15 est solution !
    Mais je suis preneur de ton raisonnement parce que j'ai essayé de raisonner aussi avec des recompositions en facteurs premiers, sans succès.

    Effectivement c'est moins direct que je ne le pensais.

    Soit $n$ un entier. Pour tout $p$ premier, notons $\nu(p)$ l'exposant de $p$ dans la factorisation de $g(n)$. Considérons un élément de $\mathfrak S_n$ d'ordre $g(n)$. Ses cycles ont des longueurs dont le p.p.c.m. vaut $g(n)$ et la somme vaut $n$. Ainsi, tout $p^{\nu(p)}$ divise la longueur d'au moins un des cycles et $\sum_{p\mid g(n)} p^{\nu(p)} \leq n$ car la longueur d'un cycle majore la somme des $p^{\nu(p)}$ qui la divisent. On peut donc construire un élément de $\mathfrak S_n$ d'ordre $g(n)$ tel que chaque cycle non trivial est de longueur $p^{\nu(p)}$ associée à un diviseur premier de $g(n)$.

    On suppose $g(n)$ impair. Soit $p$ un facteur premier de $g(n)$. On va chercher à remplacer le cycle de longueur $p^{\nu(p)}$ par deux cycles à supports disjoints de longueurs respectives $p^{\nu(p)-1}$ et $2^\alpha$, sans augmenter le support. C'est possible à condition que $2^\alpha \leq (p-1) p^{\nu(p)-1}$ et on dispose bien d'un tel $\alpha \geq 1$ tel que de plus $(p-1)p^{\nu(p)-1} < 2^{\alpha+1}$. Par maximalité de $g(n)$, qui est impair, on a nécessairement $2^\alpha p^{\nu(p)-1} < p^{\nu(p)}$, d'où $2^\alpha \leq p - 1$ et donc $p^{\nu(p)-1} < 2$. Conclusion : $\nu(p)=1$ pour tout facteur premier de $g(n)$.

    En raisonnant de même, on montre que tout facteur premier $p \geq 11$ vérifie nécessairement $p \leq 23$ (sinon on remplacerait avantageusement le cycle de longueur $p$ par deux cycles de longueurs respectives $3^2$ et $2^\alpha$). Ceci donne déjà une majoration de $g(n)$, et donc de $n$, par des constantes.

    On peut affiner la recherche avec le même type d'arguments pour aboutir à l'ensemble des solutions $\{1, 3,8,15\}$. Par exemple : si $g(n)$ est impair et n'est pas divisible par un nombre premier $p \geq 3$, alors il n'a aucun facteur premier $q > p$.
  • Merci Siméon pour cette belle démonstration qui est d'ailleurs plus simple et plus limpide que celle de l'article auquel je renvoyais. Pour ma part je voulais également "remplacer les cycles longs avantageusement" sans y parvenir. Je trouve d'ailleurs cet exercice difficile, comme beaucoup d'autres. À l'exception de quelques exercices simples et de quelques exercices "cmassiques" l'essentiel des planches me semblent tout à fait au niveau de ce qu'on est en droit d'attendre à ce concours. Tu maintiens toujours que les exercices sont simples, ou pas trop difficiles ? J'imagine que BobbyJoe sera d'accord avec moi pour dire que ces planches sont difficiles ...
  • @BJ
    quel est le plus chanceux et le moins chanceux des candidats ( en rapport avec le tirage au sors des planches)
    Le 😄 Farceur


  • Je n'en pense pas grand chose si ce n'est que les exos sont majoritairement techniques (mais n'excluent pas le fait d'avoir des idées originales).
    Ah mon avis, il ne faut pas perdre de vue que ces exos ne sont pas faits pour être résolus pendant le temps imparti (les candidats sont mesurés par le fait d'avoir des idées naturelles et de mener une réfléxion grâce à des pistes guidées ou grâce à un traitement de certains cas particuliers via des techniques de réduction).
    L'autonomie recherchée (et "précoce" dirons nous) pour mener à bien ces exercices est le gage qu'ULM cherche à recruter les meilleurs candidats en mathématiques.
  • Pour la planche 12 exo 2, j'utiliserais $(n+1)^{-n-1}n!=\int_0^1u^n(-\log u)^ndu$ , $f(u)=u^{-u}$ pour $0<u<1$ et $$\left(\sum_{n=1}^{\infty}\frac{x^n}{n^n}\right)^{1/x}=x^{1/x}\|f\|_x\to_{x\to \infty}\|f\|_{\infty}=e^{-1/e}.$$





    Pardon c'est $e^{1/e}$: merci totem.
  • @Mickaël : ces exercices sont difficiles, loin de moi l'idée de dire le contraire. Mais ceux auxquels j'ai réfléchi m'ont semblé moins infaisables qu'avant, dans le sens où les grandes idées du cours suffisent à trouver une piste valable. Et je trouve ça très bien ! Bien sûr il reste certainement quelques énoncés dont je ne saurais pas par quel bout les prendre.
  • Merci Mickaël pour avoir fait les calculs de la planche 28 exercice 1 et merci BobbyJoe, je ne connaissais pas cette inégalité d'Hoeffding.
  • Bonjour
    Que signifie le terme proportion dans la phrase suivante de Corto ?

    http://www.les-mathematiques.net/phorum/read.php?4,1841908,1845370#msg-1845370
    Un résultat bien connu nous dit que pour presque tout $x\in[0;1]$ la proportion de $9$ dans les décimales de $x$ est exactement 1/10.
  • Voici un énoncé précis de ce que je voulais dire :

    Soit $x \in [0;1]\backslash \Q$ un irrationnel, il existe une unique suite $(a_n(x))_n \in \{0; \ldots ; 9\}^{\N^*}$ telle que
    \[
    x= \sum_{k=1}^\infty \frac{a_k(x)}{10^k}.
    \]
    Les $a_n(x)$ sont communément appelés les décimales de $x$. Ici je prend uniquement les irrationnels pour éviter les blagues du genre $1 =0.999\ldots$. Ce que je dis c'est que pour presque tout $x\in [0;1]\backslash \Q$ (et donc pour presque tout $x\in[0;1]$ puisque $\lambda(\Q)=0$) on a

    \[
    \lim_{n\to \infty} \frac{1}{n} \mathrm{Card}\{k \in \{1; \ldots ; n\} : a_k(x) = 9\}=1/10.
    \]

    On trouve des démonstrations de ce résultat basées sur la loi forte des grands nombres, les systèmes dynamiques, l'équirépartition de suites etc.
  • Un grand merci pour cet énoncé précis.
  • Voici une solution pour Planche 2, exercice 1: [les-mathematiques.net]
    Je ne sais pas si elle a été donnée.
  • @P.:que signifie la norme $\|f\|_x$ ?
  • Reponse a Totem: $\|f\|_p=[\int_0^1|f(u)|^pdu]^{1/p}$
  • @P : ok merci j'ai compris.
    Ta méthode est particulièrement efficace, une fois de plus, on est au résultat en 2 signes égal ::o

    Par contre il me semble que $\|f\|_{\infty}=e^{1/e}$ ?

    As-tu une méthode aussi redoutable pour trouver un équivalent de $\sum_{n=1}^{\infty}\frac{x^n}{n^n}$ ??
  • Totem merci, la limite est $e^{1/e}$. Pour l'equivalence que tu demandes tu voudrais plus que $\log(\sum x^n/n^n)\sim_{x\to \infty} x/e?$
  • @P: je m'en serais bien contenté mais c'est faux 8-)
    Side a trouvé la réponse mais avec la méthode de Laplace qui est assez technique...
    Mais je m'en doutais un peu car $f \sim g$ n'implique pas $f^x \sim g^x $ ça se saurait !

    side écrivait : http://www.les-mathematiques.net/phorum/read.php?4,1841908,1842144#msg-1842144
    [Inutile de recopier un message présent sur le forum. Un lien suffit. AD]
  • Hein? $g(x)^{1/x}\to C \Leftrightarrow \frac{1}{x}\log g(x)\to \log C \Leftrightarrow \log g(x)\sim x\log C...$
  • Euh oui... je ne sais pas quoi répondre et j'ai du mal à me faire une idée entre ta réponse et celle de side !!
    Tu as vu que side trouve un "facteur correctif" en plus $(\pi x/2)^{1/2} \times e^{-1}$ ? Qu'en penses-tu ?
    Side si tu repasses par là !:-)
  • Rendez-vous à la page 62 d'Asymptotic methods in analysis' NG de Bruijn, Dover 1981, appliquée à la fonction $h(u)=-u\log u$ sur $]0,1[$ pour vérifier qu'en effet ce qu’écrit side est correct.
  • Je n'ai pas ce livre et ce n'est pas au fond de mon bled que je vais le trouver :-D

    Du coup, question : pourquoi ce que tu as écrit est faux ?? Ou plutôt que faux, on ne peut pas composer comme ça par l'exponentielle à la dernière relation d'équivalence je pense ?
  • Rien de faux dans les ecrits de side et de ton serviteur.
  • Il me semble qu'il y a un problème dans ce message http://www.les-mathematiques.net/phorum/read.php?4,1841908,1853176#msg-1853176, car on a un probleme avec $g(n)=n!$( edit une coquille g(n)=n²) c'est à dire $g(n)^{\frac 1n}\to 1$ mais $\ln(g(n))\not \sim n\ln(1)$
    Je n'ai pas suivi la discussion, donc peut-être je suis passé à coté du contexte. :-D
    Le 😄 Farceur


  • @: pas faux mais incomplet...que signifient tes 3 points de suspension ?:-D
  • $(n!)^{1/n}\to 1?$ Tiens tiens.
Connectez-vous ou Inscrivez-vous pour répondre.
Success message!